Equicontinuous sequences of functions vs. continuous functions

Click For Summary
The discussion revolves around verifying a solution related to equicontinuous sequences of functions and continuous functions. Participants seek clarification on the notation used, particularly the meaning of the subscript 'k' on delta, which is intended to indicate a specific delta for a given function and epsilon. Suggestions are made to improve clarity by labeling the delta as delta_n instead of using the ambiguous subscript k. The conversation emphasizes the importance of precise notation in mathematical discussions. Overall, the focus is on enhancing understanding and communication regarding the problem at hand.
jdinatale
Messages
153
Reaction score
0
Hello,

below I have the problem and solution typed in Latex. For the first part, I just want someone to verify if I am correct.

For the second part, I need guidance in the right direction

math1.png

 
Physics news on Phys.org
I think you've got the right idea for a), but your description is a little muddled. Like what is the k subscript on your delta supposed to mean? For b) think about how you would go about choosing a delta to satisfy |g_n(1)-g_n(x)|<\frac{1}{2}.
 
Dick said:
I think you've got the right idea for a), but your description is a little muddled. Like what is the k subscript on your delta supposed to mean? For b) think about how you would go about choosing a delta to satisfy |g_n(1)-g_n(x)|<\frac{1}{2}.

Thank you. The k on the delta is just to show that for a given f_n and epsilon, you will get a particular delta which I call delta subscript k. I guess I could remove that.
 
jdinatale said:
Thank you. The k on the delta is just to show that for a given f_n and epsilon, you will get a particular delta which I call delta subscript k. I guess I could remove that.

Wouldn't it make more sense that if delta depends on f_n, that that particular delta should be labeled delta_n, instead of the meaningless subscript k? That's what I'm talking about with notation.
 
Question: A clock's minute hand has length 4 and its hour hand has length 3. What is the distance between the tips at the moment when it is increasing most rapidly?(Putnam Exam Question) Answer: Making assumption that both the hands moves at constant angular velocities, the answer is ## \sqrt{7} .## But don't you think this assumption is somewhat doubtful and wrong?

Similar threads

  • · Replies 27 ·
Replies
27
Views
2K
  • · Replies 40 ·
2
Replies
40
Views
4K
  • · Replies 13 ·
Replies
13
Views
2K
Replies
7
Views
1K
Replies
3
Views
2K
  • · Replies 1 ·
Replies
1
Views
1K
  • · Replies 4 ·
Replies
4
Views
2K
  • · Replies 2 ·
Replies
2
Views
2K
  • · Replies 7 ·
Replies
7
Views
2K
  • · Replies 8 ·
Replies
8
Views
4K